Was steigt schneller? 1337^n^3 oder (n+1)!

Neue Frage »

hatschielgesundheit Auf diesen Beitrag antworten »
Was steigt schneller? 1337^n^3 oder (n+1)!
Meine Frage:
Welche Funktion steigt für große n schneller und wieso?

1337^n^3 oder (n+1)!

?


Meine Ideen:
Normalerweise steigt ja die Fakultät immer schneller aber hier bin ich mir nicht sicher, bzw. kann es nicht zeigen.
Nofeykx Auf diesen Beitrag antworten »

Bitte erstmal eindeutig klammern.

Variante 1:

Variante 2:
hatschielgesundheit Auf diesen Beitrag antworten »

Variante 1, sonst hätte ich ja eine Klammer setzen müssen :-P



Meine Idee: Per Induktion mit Voraussetzung (n+1)! = 1337^n^3

Nach dem Ind. Schritt und kürzen folgt dann doch nur noch, dass man n+2 auf der einen Seite hat und auf der anderen 1337^(3n^2+3n+1), oder? Aber wie zeige ich, dass der letzte Term größer als n+2 ist?
Nofeykx Auf diesen Beitrag antworten »

Hi, ich weiß zwar grad nicht, was du mit

Zitat:
Per Induktion mit Voraussetzung (n+1)! = 1337^n^3


meinst, das ist aber auch nicht so wichtig, denn das richtige hast du offenbar rausbekommen.

Erhöht sich n bei der Fakultät um 1, steigt der Wert des Ausdrucks um n+2 und analog bei dem anderen Teil.

Zitat:
Aber wie zeige ich, dass der letzte Term größer als n+2 ist?


Nun, das eine ist ein Polynom und das andere etwas, das noch schneller wächst, als eine Exponentialfunktion. Ob du voraussetzen möchtest, dass die Exponentialfunktion schneller wächst und du damit fertig bist, musst du entscheiden.
Nofeykx Auf diesen Beitrag antworten »

Ich meinte natürlich oben, dass die Fakultät dann um den Faktor n+2 ansteigt, nicht einfach so n+2 Big Laugh
hatschielgesundheit Auf diesen Beitrag antworten »

Nein, ich habe es leider noch nicht raus unglücklich

Es scheint ja so, als ob 1337^n^3 schneller wächst als die Fakultät. Ich kann aber trotz meinem Anfang nicht genau erklären wieso das so ist.

bei der Induktion habe ich einfach mal (n+1)!=1337^n^3 angenommen, um im Schritt zu zeigen, dass das eine schneller als das andere wächst. Aber da bin ich ja noch nicht ganz fertig...
 
 
Nofeykx Auf diesen Beitrag antworten »

Zitat:
bei der Induktion habe ich einfach mal (n+1)!=1337^n^3 angenommen, um im Schritt zu zeigen, dass das eine schneller als das andere wächst.


Achso! Nein, das geht nicht. Du kannst das nicht einfach annehmen, es stimmt ja nicht.

Alternative:

Definiere dir eine Folge:



Wenn du jetzt zeigst, dass diese Folge bestimmt divergiert hast dus.

Dafür kannst du zB den Quotient zweier aufeinanderfolgender Folgenglieder bestimmen. Damit bekommst du dann genau, was du schon hast.

Für die Divergenz von reicht schon, dass dieser Quotient für fast alle n größer als 1 ist.
hatschielgesundheit Auf diesen Beitrag antworten »

(gelöscht) --> verstanden. smile Vielen Dank smile
hatschielgesundheit Auf diesen Beitrag antworten »

(gelöscht)
Neue Frage »
Antworten »



Verwandte Themen

Die Beliebtesten »
Die Größten »
Die Neuesten »